PASQUETTA 2015

Teorema di Banach–Kuratowski
Cominciamo con una notazione: se S = (n1 , n2 , . . . ) e T = (k1 , k2 , . . . )
sono due successioni di interi strettamente positivi, scriveremo S T se, per
ogni i, ni ≤ ki . Quando questa propriet`a non `e verificata scriveremo S T .
Lemma 0.1. Esiste un insieme F che ha la cardinalit`a del continuo, i
cui punti sono successioni di interi positivi, e tale
ognisuccessio che, per
ne S (anche non appartenente ad F ) l’insieme T ∈ F T S `e al pi`
u
numerabile.
Dimostrazione. Consideriamo l’insieme delle successioni di interi positivi (che
ha cardinalit`a del continuo) e ordiniamo i suoi elementi S0 , S1 , .., Sa , .. a < ω1
(dove ω1 `e il primo ordinale non numerabile).
Presa una successione T1 , T2 , . . . si pu`o costruire una nuova successione
S tale che S Tn ∀n .
Per ogni ordinale a < ω1 scegliamo ξa tale che, se b < a, Sξa Sb e
Sξa 6= Sξb . L’insieme F `e l’insieme delle successioni Sξa , a < ω1 che ha la
cardinalit`a del continuo.
L’insieme F
ha la propriet`
a voluta poich´e, se Sξa Sb necessariamente
a ≤ b (quindi T ∈ F T Sb `e al pi`
u numerabile).
Lemma 0.2. Sia E un insieme con la cardinalit`a del continuo: si pu`o
decomporre
E = A11 ∪ A12 ∪ . . .
E = A21 ∪ A22 ∪ . . .
............
E = Ai1 ∪ Ai2 ∪ . . .
dove, per ogni i, Ai1 , Ai2 , . . . `e una partizione di E e, presa una qualunque
successione di interi positivi k1 , k2 , . . .
+∞
\
Ai1 ∪ Ai2 ∪ · · · ∪ Aiki
i=1
`e al pi`
u numerabile.
Dimostrazione. Stabiliamo una corrispondenza biunivoca x ↔ Tx tra i punti
di E ed i punti di F : scriviamo Tx = nx1 , nx2 , . . . .
Definiamo gli insiemi Aik in questo modo: x ∈ Aik se nxi = k . In questo
modo gli insiemi Ai1 , Ai2 , . . . formano effettivamente una partizione di E.
1
Notiamo ancora che x ∈ Ai1 ∪ · · · ∪ Aiki se nxi ≤ ki , quindi posto S =
k1 , k2 , . . .
+∞
\
Ai1 ∪ Ai2 ∪ · · · ∪ Aiki = x ∈ E Tx S
i=1
`e al pi`
u numerabile.
Teorema 0.3 (Banach-Kuratowski). Sia E un insieme con la cardinalit`a
del continuo: non pu`o esistere una probabilit`a diffusa m definita su tutti i
sottinsiemi di E.
Dimostrazione. Supponiamo l’esistenza di questa m e proviamo che si arriva
a un assurdo. Per ogni i ≥ 1 , esiste ki tale che, posto Ri = Aiki +1 ∪ Aiki +2 ∪
Aiki +3 ∪ . . . si abbia m(Ri ) ≤ 2−(i+1) .
c
S
S
+∞ i
+∞ i
R
≥ 1/2 .
≤
1/2
e
di
conseguenza
m
R
Dunque m
i=1
i=1
Questo per`o `e impossibile poich´e
+∞
[
i=1
R
i
c
=
+∞
\
Ai1 ∪ Ai2 ∪ · · · ∪ Aiki
i=1
`e al pi`
u numerabile.
2